LSAT and Law School Admissions Forum

Get expert LSAT preparation and law school admissions advice from PowerScore Test Preparation.

 Administrator
PowerScore Staff
  • PowerScore Staff
  • Posts: 8929
  • Joined: Feb 02, 2011
|
#104142
Complete Question Explanation

Justify the Conclusion. The correct answer choice is (E).

Answer choice (A):

Answer choice (B):

Answer choice (C):

Answer choice (D):

Answer choice (E): This is the correct answer choice.

This explanation is still in progress. Please post any questions below!
 alphonse92m@gmail.com
  • Posts: 5
  • Joined: Feb 14, 2021
|
#105628
Can someone explain why E is the answer and not B?
User avatar
 Jeff Wren
PowerScore Staff
  • PowerScore Staff
  • Posts: 451
  • Joined: Oct 19, 2022
|
#105646
Hi Alphonse,

Let's start by rearranging the premises and conclusion.

P: Effective dramatic performances do not detract from the audience’s appreciation of a play.
P: Calling the audience’s attention to the fact that it is a performance would make it more difficult for the audience to empathize with the character the actor is playing.

C: An effective acting performance does not call the audience’s attention to the fact that it is a performance.

The gap in the argument is between making it more difficult for the audience to empathize with the actor’s character and detracting from the audience’s appreciation of the play. Answer E properly connects those terms and justifies the argument when added to the premises.

Answer B is basically saying if a performance doesn't call attention, then it won't detract. This is a Mistaken Reversal of what we would need to justify the argument. Notice that doesn't call attention (the term in our conclusion) is the sufficient condition here, which is backwards.

Get the most out of your LSAT Prep Plus subscription.

Analyze and track your performance with our Testing and Analytics Package.